Word Problem Involving Generating Functions












5












$begingroup$



Question



Let $(a_n)_{ngeq 0}$ be an increasing sequence of non-negative integers such that every non-negative integer can be expressed uniquely in the form $a_i+2a_j+4a_k$ where $i,j,k$ are not necessarily distinct. Compute $a_{1998}$.




My attempt



Put $A(x)=sum_{jgeq 0}x^{a_j}$ and translate the given condition in the language of generating functions to yield that
$$
frac{1}{1-x}=A(x)A(x^2)A(x^4).tag{0}
$$

Substituting $x^2$ in place of $x$ in $(0)$ yields that
$$
frac{A(x)A(x^2)A(x^4)}{1+x}=frac{1}{1-x^2}=A(x^2)A(x^4)A(x^8)tag{1}
$$

So $A(x)=(1+x)A(x^8)$. Iterating we get that
$$
A(x)=prod_{jgeq0}(1+x^{8^j})=sum_{jgeq0}x^{a_j}.tag{2}
$$

My Problem



But I am do not know how to deduce $a_{1998}$ from $(2)$. Any help is appreciated.










share|cite|improve this question









$endgroup$

















    5












    $begingroup$



    Question



    Let $(a_n)_{ngeq 0}$ be an increasing sequence of non-negative integers such that every non-negative integer can be expressed uniquely in the form $a_i+2a_j+4a_k$ where $i,j,k$ are not necessarily distinct. Compute $a_{1998}$.




    My attempt



    Put $A(x)=sum_{jgeq 0}x^{a_j}$ and translate the given condition in the language of generating functions to yield that
    $$
    frac{1}{1-x}=A(x)A(x^2)A(x^4).tag{0}
    $$

    Substituting $x^2$ in place of $x$ in $(0)$ yields that
    $$
    frac{A(x)A(x^2)A(x^4)}{1+x}=frac{1}{1-x^2}=A(x^2)A(x^4)A(x^8)tag{1}
    $$

    So $A(x)=(1+x)A(x^8)$. Iterating we get that
    $$
    A(x)=prod_{jgeq0}(1+x^{8^j})=sum_{jgeq0}x^{a_j}.tag{2}
    $$

    My Problem



    But I am do not know how to deduce $a_{1998}$ from $(2)$. Any help is appreciated.










    share|cite|improve this question









    $endgroup$















      5












      5








      5


      1



      $begingroup$



      Question



      Let $(a_n)_{ngeq 0}$ be an increasing sequence of non-negative integers such that every non-negative integer can be expressed uniquely in the form $a_i+2a_j+4a_k$ where $i,j,k$ are not necessarily distinct. Compute $a_{1998}$.




      My attempt



      Put $A(x)=sum_{jgeq 0}x^{a_j}$ and translate the given condition in the language of generating functions to yield that
      $$
      frac{1}{1-x}=A(x)A(x^2)A(x^4).tag{0}
      $$

      Substituting $x^2$ in place of $x$ in $(0)$ yields that
      $$
      frac{A(x)A(x^2)A(x^4)}{1+x}=frac{1}{1-x^2}=A(x^2)A(x^4)A(x^8)tag{1}
      $$

      So $A(x)=(1+x)A(x^8)$. Iterating we get that
      $$
      A(x)=prod_{jgeq0}(1+x^{8^j})=sum_{jgeq0}x^{a_j}.tag{2}
      $$

      My Problem



      But I am do not know how to deduce $a_{1998}$ from $(2)$. Any help is appreciated.










      share|cite|improve this question









      $endgroup$





      Question



      Let $(a_n)_{ngeq 0}$ be an increasing sequence of non-negative integers such that every non-negative integer can be expressed uniquely in the form $a_i+2a_j+4a_k$ where $i,j,k$ are not necessarily distinct. Compute $a_{1998}$.




      My attempt



      Put $A(x)=sum_{jgeq 0}x^{a_j}$ and translate the given condition in the language of generating functions to yield that
      $$
      frac{1}{1-x}=A(x)A(x^2)A(x^4).tag{0}
      $$

      Substituting $x^2$ in place of $x$ in $(0)$ yields that
      $$
      frac{A(x)A(x^2)A(x^4)}{1+x}=frac{1}{1-x^2}=A(x^2)A(x^4)A(x^8)tag{1}
      $$

      So $A(x)=(1+x)A(x^8)$. Iterating we get that
      $$
      A(x)=prod_{jgeq0}(1+x^{8^j})=sum_{jgeq0}x^{a_j}.tag{2}
      $$

      My Problem



      But I am do not know how to deduce $a_{1998}$ from $(2)$. Any help is appreciated.







      combinatorics discrete-mathematics generating-functions






      share|cite|improve this question













      share|cite|improve this question











      share|cite|improve this question




      share|cite|improve this question










      asked Dec 25 '18 at 23:10









      Foobaz JohnFoobaz John

      22.9k41552




      22.9k41552






















          3 Answers
          3






          active

          oldest

          votes


















          2












          $begingroup$

          Nice approach! To finish, first check that your expression is compatible with the condition that the $a_j$ strictly increase with $j$. You can do this by proving that all coefficients in the finite product
          $$
          P_k(x)=prod_{j=0}^kleft(1+x^{8^j}right)=(1+x)left(1+x^8right)left(1+x^{64}right)ldotsleft(1+x^{8^k}right)
          $$

          equal $0$ or $1$, using induction on $k$. (You will use that $1+8+64+ldots+8^k<8^{k+1}$.). You will, in fact, have accomplished more, in having shown that the $a_j$ are precisely the non-negative integers whose base-$8$ representation uses only the digits $0$ and $1$.



          So now you need the $1998^text{th}$ such integer (with $0$ being the zeroth). The first will be $1_8=1$; the second will be $10_8=8$; the third will be $11_8=9$. You should now see the the prescription in jmerry's answer applies.






          share|cite|improve this answer









          $endgroup$





















            3












            $begingroup$

            I'd rather go low-tech on this one - find a few values, then look for a pattern. We clearly have $a_0=0$ as $0+2cdot 0+4cdot 0$ is the only way to get zero, then $a_1=1$. We get everything up to $7$ that way, but $8$ is too big, so $a_2=8$. What's next? Well, we can't get $9$; we need a copy of $8$ in there, but the remaining $1$ can't be found from the $2$ and $4$ multiples left over. Thus $a_3=9$.



            After that, it's a long time - everything up to $63=9+2cdot 9+4cdot 9$ works. That leaves $x_4=64$ - and, similar to how we needed $9$, we need $x_5=65$.



            OK, I see the pattern now - we take everything with a base-8 expansion that consists entirely of zeros and ones. Checking... we can build a sum for an arbitrary nonnegative integer "digit" by "digit", using what we already know about building $0$ through $7$ out of zeros and ones. Package everything with the same multiplier together, and there it is.



            All right, how do we extract an element of the sequence of a particular index? Write it down in base $2$, read it out in base $8$. For $1998$, that's
            $$8^{10}+8^9+8^8+8^7+8^6+8^3+8^2+8^1 = 1227096648$$






            share|cite|improve this answer









            $endgroup$





















              1












              $begingroup$

              If your formula for A(x) is correct,
              then $a_j$ is the sum of distinct powers of eight.
              But 1998 is congruent to 6 mod 8
              And so cannot be written in this way.



              Therefore $a_{1998} = 0$.






              share|cite|improve this answer









              $endgroup$









              • 1




                $begingroup$
                $a_{1998}$ represents the $1999^text{th}$ power in the sum that has coefficient $1$. (All other powers have coefficient $0$.)
                $endgroup$
                – Will Orrick
                Dec 28 '18 at 12:46










              • $begingroup$
                It's $a_j$ that is a sum of distinct powers of $8$, not $j$, so it doesn't matter that $jequiv6pmod8$.
                $endgroup$
                – Will Orrick
                Dec 30 '18 at 18:59














              Your Answer





              StackExchange.ifUsing("editor", function () {
              return StackExchange.using("mathjaxEditing", function () {
              StackExchange.MarkdownEditor.creationCallbacks.add(function (editor, postfix) {
              StackExchange.mathjaxEditing.prepareWmdForMathJax(editor, postfix, [["$", "$"], ["\\(","\\)"]]);
              });
              });
              }, "mathjax-editing");

              StackExchange.ready(function() {
              var channelOptions = {
              tags: "".split(" "),
              id: "69"
              };
              initTagRenderer("".split(" "), "".split(" "), channelOptions);

              StackExchange.using("externalEditor", function() {
              // Have to fire editor after snippets, if snippets enabled
              if (StackExchange.settings.snippets.snippetsEnabled) {
              StackExchange.using("snippets", function() {
              createEditor();
              });
              }
              else {
              createEditor();
              }
              });

              function createEditor() {
              StackExchange.prepareEditor({
              heartbeatType: 'answer',
              autoActivateHeartbeat: false,
              convertImagesToLinks: true,
              noModals: true,
              showLowRepImageUploadWarning: true,
              reputationToPostImages: 10,
              bindNavPrevention: true,
              postfix: "",
              imageUploader: {
              brandingHtml: "Powered by u003ca class="icon-imgur-white" href="https://imgur.com/"u003eu003c/au003e",
              contentPolicyHtml: "User contributions licensed under u003ca href="https://creativecommons.org/licenses/by-sa/3.0/"u003ecc by-sa 3.0 with attribution requiredu003c/au003e u003ca href="https://stackoverflow.com/legal/content-policy"u003e(content policy)u003c/au003e",
              allowUrls: true
              },
              noCode: true, onDemand: true,
              discardSelector: ".discard-answer"
              ,immediatelyShowMarkdownHelp:true
              });


              }
              });














              draft saved

              draft discarded


















              StackExchange.ready(
              function () {
              StackExchange.openid.initPostLogin('.new-post-login', 'https%3a%2f%2fmath.stackexchange.com%2fquestions%2f3052506%2fword-problem-involving-generating-functions%23new-answer', 'question_page');
              }
              );

              Post as a guest















              Required, but never shown

























              3 Answers
              3






              active

              oldest

              votes








              3 Answers
              3






              active

              oldest

              votes









              active

              oldest

              votes






              active

              oldest

              votes









              2












              $begingroup$

              Nice approach! To finish, first check that your expression is compatible with the condition that the $a_j$ strictly increase with $j$. You can do this by proving that all coefficients in the finite product
              $$
              P_k(x)=prod_{j=0}^kleft(1+x^{8^j}right)=(1+x)left(1+x^8right)left(1+x^{64}right)ldotsleft(1+x^{8^k}right)
              $$

              equal $0$ or $1$, using induction on $k$. (You will use that $1+8+64+ldots+8^k<8^{k+1}$.). You will, in fact, have accomplished more, in having shown that the $a_j$ are precisely the non-negative integers whose base-$8$ representation uses only the digits $0$ and $1$.



              So now you need the $1998^text{th}$ such integer (with $0$ being the zeroth). The first will be $1_8=1$; the second will be $10_8=8$; the third will be $11_8=9$. You should now see the the prescription in jmerry's answer applies.






              share|cite|improve this answer









              $endgroup$


















                2












                $begingroup$

                Nice approach! To finish, first check that your expression is compatible with the condition that the $a_j$ strictly increase with $j$. You can do this by proving that all coefficients in the finite product
                $$
                P_k(x)=prod_{j=0}^kleft(1+x^{8^j}right)=(1+x)left(1+x^8right)left(1+x^{64}right)ldotsleft(1+x^{8^k}right)
                $$

                equal $0$ or $1$, using induction on $k$. (You will use that $1+8+64+ldots+8^k<8^{k+1}$.). You will, in fact, have accomplished more, in having shown that the $a_j$ are precisely the non-negative integers whose base-$8$ representation uses only the digits $0$ and $1$.



                So now you need the $1998^text{th}$ such integer (with $0$ being the zeroth). The first will be $1_8=1$; the second will be $10_8=8$; the third will be $11_8=9$. You should now see the the prescription in jmerry's answer applies.






                share|cite|improve this answer









                $endgroup$
















                  2












                  2








                  2





                  $begingroup$

                  Nice approach! To finish, first check that your expression is compatible with the condition that the $a_j$ strictly increase with $j$. You can do this by proving that all coefficients in the finite product
                  $$
                  P_k(x)=prod_{j=0}^kleft(1+x^{8^j}right)=(1+x)left(1+x^8right)left(1+x^{64}right)ldotsleft(1+x^{8^k}right)
                  $$

                  equal $0$ or $1$, using induction on $k$. (You will use that $1+8+64+ldots+8^k<8^{k+1}$.). You will, in fact, have accomplished more, in having shown that the $a_j$ are precisely the non-negative integers whose base-$8$ representation uses only the digits $0$ and $1$.



                  So now you need the $1998^text{th}$ such integer (with $0$ being the zeroth). The first will be $1_8=1$; the second will be $10_8=8$; the third will be $11_8=9$. You should now see the the prescription in jmerry's answer applies.






                  share|cite|improve this answer









                  $endgroup$



                  Nice approach! To finish, first check that your expression is compatible with the condition that the $a_j$ strictly increase with $j$. You can do this by proving that all coefficients in the finite product
                  $$
                  P_k(x)=prod_{j=0}^kleft(1+x^{8^j}right)=(1+x)left(1+x^8right)left(1+x^{64}right)ldotsleft(1+x^{8^k}right)
                  $$

                  equal $0$ or $1$, using induction on $k$. (You will use that $1+8+64+ldots+8^k<8^{k+1}$.). You will, in fact, have accomplished more, in having shown that the $a_j$ are precisely the non-negative integers whose base-$8$ representation uses only the digits $0$ and $1$.



                  So now you need the $1998^text{th}$ such integer (with $0$ being the zeroth). The first will be $1_8=1$; the second will be $10_8=8$; the third will be $11_8=9$. You should now see the the prescription in jmerry's answer applies.







                  share|cite|improve this answer












                  share|cite|improve this answer



                  share|cite|improve this answer










                  answered Dec 28 '18 at 13:45









                  Will OrrickWill Orrick

                  13.7k13461




                  13.7k13461























                      3












                      $begingroup$

                      I'd rather go low-tech on this one - find a few values, then look for a pattern. We clearly have $a_0=0$ as $0+2cdot 0+4cdot 0$ is the only way to get zero, then $a_1=1$. We get everything up to $7$ that way, but $8$ is too big, so $a_2=8$. What's next? Well, we can't get $9$; we need a copy of $8$ in there, but the remaining $1$ can't be found from the $2$ and $4$ multiples left over. Thus $a_3=9$.



                      After that, it's a long time - everything up to $63=9+2cdot 9+4cdot 9$ works. That leaves $x_4=64$ - and, similar to how we needed $9$, we need $x_5=65$.



                      OK, I see the pattern now - we take everything with a base-8 expansion that consists entirely of zeros and ones. Checking... we can build a sum for an arbitrary nonnegative integer "digit" by "digit", using what we already know about building $0$ through $7$ out of zeros and ones. Package everything with the same multiplier together, and there it is.



                      All right, how do we extract an element of the sequence of a particular index? Write it down in base $2$, read it out in base $8$. For $1998$, that's
                      $$8^{10}+8^9+8^8+8^7+8^6+8^3+8^2+8^1 = 1227096648$$






                      share|cite|improve this answer









                      $endgroup$


















                        3












                        $begingroup$

                        I'd rather go low-tech on this one - find a few values, then look for a pattern. We clearly have $a_0=0$ as $0+2cdot 0+4cdot 0$ is the only way to get zero, then $a_1=1$. We get everything up to $7$ that way, but $8$ is too big, so $a_2=8$. What's next? Well, we can't get $9$; we need a copy of $8$ in there, but the remaining $1$ can't be found from the $2$ and $4$ multiples left over. Thus $a_3=9$.



                        After that, it's a long time - everything up to $63=9+2cdot 9+4cdot 9$ works. That leaves $x_4=64$ - and, similar to how we needed $9$, we need $x_5=65$.



                        OK, I see the pattern now - we take everything with a base-8 expansion that consists entirely of zeros and ones. Checking... we can build a sum for an arbitrary nonnegative integer "digit" by "digit", using what we already know about building $0$ through $7$ out of zeros and ones. Package everything with the same multiplier together, and there it is.



                        All right, how do we extract an element of the sequence of a particular index? Write it down in base $2$, read it out in base $8$. For $1998$, that's
                        $$8^{10}+8^9+8^8+8^7+8^6+8^3+8^2+8^1 = 1227096648$$






                        share|cite|improve this answer









                        $endgroup$
















                          3












                          3








                          3





                          $begingroup$

                          I'd rather go low-tech on this one - find a few values, then look for a pattern. We clearly have $a_0=0$ as $0+2cdot 0+4cdot 0$ is the only way to get zero, then $a_1=1$. We get everything up to $7$ that way, but $8$ is too big, so $a_2=8$. What's next? Well, we can't get $9$; we need a copy of $8$ in there, but the remaining $1$ can't be found from the $2$ and $4$ multiples left over. Thus $a_3=9$.



                          After that, it's a long time - everything up to $63=9+2cdot 9+4cdot 9$ works. That leaves $x_4=64$ - and, similar to how we needed $9$, we need $x_5=65$.



                          OK, I see the pattern now - we take everything with a base-8 expansion that consists entirely of zeros and ones. Checking... we can build a sum for an arbitrary nonnegative integer "digit" by "digit", using what we already know about building $0$ through $7$ out of zeros and ones. Package everything with the same multiplier together, and there it is.



                          All right, how do we extract an element of the sequence of a particular index? Write it down in base $2$, read it out in base $8$. For $1998$, that's
                          $$8^{10}+8^9+8^8+8^7+8^6+8^3+8^2+8^1 = 1227096648$$






                          share|cite|improve this answer









                          $endgroup$



                          I'd rather go low-tech on this one - find a few values, then look for a pattern. We clearly have $a_0=0$ as $0+2cdot 0+4cdot 0$ is the only way to get zero, then $a_1=1$. We get everything up to $7$ that way, but $8$ is too big, so $a_2=8$. What's next? Well, we can't get $9$; we need a copy of $8$ in there, but the remaining $1$ can't be found from the $2$ and $4$ multiples left over. Thus $a_3=9$.



                          After that, it's a long time - everything up to $63=9+2cdot 9+4cdot 9$ works. That leaves $x_4=64$ - and, similar to how we needed $9$, we need $x_5=65$.



                          OK, I see the pattern now - we take everything with a base-8 expansion that consists entirely of zeros and ones. Checking... we can build a sum for an arbitrary nonnegative integer "digit" by "digit", using what we already know about building $0$ through $7$ out of zeros and ones. Package everything with the same multiplier together, and there it is.



                          All right, how do we extract an element of the sequence of a particular index? Write it down in base $2$, read it out in base $8$. For $1998$, that's
                          $$8^{10}+8^9+8^8+8^7+8^6+8^3+8^2+8^1 = 1227096648$$







                          share|cite|improve this answer












                          share|cite|improve this answer



                          share|cite|improve this answer










                          answered Dec 26 '18 at 0:31









                          jmerryjmerry

                          17k11633




                          17k11633























                              1












                              $begingroup$

                              If your formula for A(x) is correct,
                              then $a_j$ is the sum of distinct powers of eight.
                              But 1998 is congruent to 6 mod 8
                              And so cannot be written in this way.



                              Therefore $a_{1998} = 0$.






                              share|cite|improve this answer









                              $endgroup$









                              • 1




                                $begingroup$
                                $a_{1998}$ represents the $1999^text{th}$ power in the sum that has coefficient $1$. (All other powers have coefficient $0$.)
                                $endgroup$
                                – Will Orrick
                                Dec 28 '18 at 12:46










                              • $begingroup$
                                It's $a_j$ that is a sum of distinct powers of $8$, not $j$, so it doesn't matter that $jequiv6pmod8$.
                                $endgroup$
                                – Will Orrick
                                Dec 30 '18 at 18:59


















                              1












                              $begingroup$

                              If your formula for A(x) is correct,
                              then $a_j$ is the sum of distinct powers of eight.
                              But 1998 is congruent to 6 mod 8
                              And so cannot be written in this way.



                              Therefore $a_{1998} = 0$.






                              share|cite|improve this answer









                              $endgroup$









                              • 1




                                $begingroup$
                                $a_{1998}$ represents the $1999^text{th}$ power in the sum that has coefficient $1$. (All other powers have coefficient $0$.)
                                $endgroup$
                                – Will Orrick
                                Dec 28 '18 at 12:46










                              • $begingroup$
                                It's $a_j$ that is a sum of distinct powers of $8$, not $j$, so it doesn't matter that $jequiv6pmod8$.
                                $endgroup$
                                – Will Orrick
                                Dec 30 '18 at 18:59
















                              1












                              1








                              1





                              $begingroup$

                              If your formula for A(x) is correct,
                              then $a_j$ is the sum of distinct powers of eight.
                              But 1998 is congruent to 6 mod 8
                              And so cannot be written in this way.



                              Therefore $a_{1998} = 0$.






                              share|cite|improve this answer









                              $endgroup$



                              If your formula for A(x) is correct,
                              then $a_j$ is the sum of distinct powers of eight.
                              But 1998 is congruent to 6 mod 8
                              And so cannot be written in this way.



                              Therefore $a_{1998} = 0$.







                              share|cite|improve this answer












                              share|cite|improve this answer



                              share|cite|improve this answer










                              answered Dec 25 '18 at 23:25









                              marty cohenmarty cohen

                              75.1k549130




                              75.1k549130








                              • 1




                                $begingroup$
                                $a_{1998}$ represents the $1999^text{th}$ power in the sum that has coefficient $1$. (All other powers have coefficient $0$.)
                                $endgroup$
                                – Will Orrick
                                Dec 28 '18 at 12:46










                              • $begingroup$
                                It's $a_j$ that is a sum of distinct powers of $8$, not $j$, so it doesn't matter that $jequiv6pmod8$.
                                $endgroup$
                                – Will Orrick
                                Dec 30 '18 at 18:59
















                              • 1




                                $begingroup$
                                $a_{1998}$ represents the $1999^text{th}$ power in the sum that has coefficient $1$. (All other powers have coefficient $0$.)
                                $endgroup$
                                – Will Orrick
                                Dec 28 '18 at 12:46










                              • $begingroup$
                                It's $a_j$ that is a sum of distinct powers of $8$, not $j$, so it doesn't matter that $jequiv6pmod8$.
                                $endgroup$
                                – Will Orrick
                                Dec 30 '18 at 18:59










                              1




                              1




                              $begingroup$
                              $a_{1998}$ represents the $1999^text{th}$ power in the sum that has coefficient $1$. (All other powers have coefficient $0$.)
                              $endgroup$
                              – Will Orrick
                              Dec 28 '18 at 12:46




                              $begingroup$
                              $a_{1998}$ represents the $1999^text{th}$ power in the sum that has coefficient $1$. (All other powers have coefficient $0$.)
                              $endgroup$
                              – Will Orrick
                              Dec 28 '18 at 12:46












                              $begingroup$
                              It's $a_j$ that is a sum of distinct powers of $8$, not $j$, so it doesn't matter that $jequiv6pmod8$.
                              $endgroup$
                              – Will Orrick
                              Dec 30 '18 at 18:59






                              $begingroup$
                              It's $a_j$ that is a sum of distinct powers of $8$, not $j$, so it doesn't matter that $jequiv6pmod8$.
                              $endgroup$
                              – Will Orrick
                              Dec 30 '18 at 18:59




















                              draft saved

                              draft discarded




















































                              Thanks for contributing an answer to Mathematics Stack Exchange!


                              • Please be sure to answer the question. Provide details and share your research!

                              But avoid



                              • Asking for help, clarification, or responding to other answers.

                              • Making statements based on opinion; back them up with references or personal experience.


                              Use MathJax to format equations. MathJax reference.


                              To learn more, see our tips on writing great answers.




                              draft saved


                              draft discarded














                              StackExchange.ready(
                              function () {
                              StackExchange.openid.initPostLogin('.new-post-login', 'https%3a%2f%2fmath.stackexchange.com%2fquestions%2f3052506%2fword-problem-involving-generating-functions%23new-answer', 'question_page');
                              }
                              );

                              Post as a guest















                              Required, but never shown





















































                              Required, but never shown














                              Required, but never shown












                              Required, but never shown







                              Required, but never shown

































                              Required, but never shown














                              Required, but never shown












                              Required, but never shown







                              Required, but never shown







                              Popular posts from this blog

                              Le Mesnil-Réaume

                              Ida-Boy-Ed-Garten

                              web3.py web3.isConnected() returns false always